Last visit was: 25 Apr 2024, 03:52 It is currently 25 Apr 2024, 03:52

Close
GMAT Club Daily Prep
Thank you for using the timer - this advanced tool can estimate your performance and suggest more practice questions. We have subscribed you to Daily Prep Questions via email.

Customized
for You

we will pick new questions that match your level based on your Timer History

Track
Your Progress

every week, we’ll send you an estimated GMAT score based on your performance

Practice
Pays

we will pick new questions that match your level based on your Timer History
Not interested in getting valuable practice questions and articles delivered to your email? No problem, unsubscribe here.
Close
Request Expert Reply
Confirm Cancel
SORT BY:
Date
Math Expert
Joined: 02 Sep 2009
Posts: 92912
Own Kudos [?]: 618902 [49]
Given Kudos: 81595
Send PM
Most Helpful Reply
Math Expert
Joined: 02 Sep 2009
Posts: 92912
Own Kudos [?]: 618902 [29]
Given Kudos: 81595
Send PM
General Discussion
avatar
Intern
Intern
Joined: 11 Aug 2016
Posts: 6
Own Kudos [?]: 7 [0]
Given Kudos: 14
Location: United States (PA)
Concentration: Finance, General Management
GMAT 1: 710 Q48 V40
GPA: 2.42
WE:Other (Mutual Funds and Brokerage)
Send PM
Intern
Intern
Joined: 18 Nov 2011
Posts: 3
Own Kudos [?]: 2 [1]
Given Kudos: 1
Send PM
Re: M01-08 [#permalink]
1
Kudos
Since this is a PS question. shouldn't the answer choices be either ascending or descending?
Founder
Founder
Joined: 04 Dec 2002
Posts: 37305
Own Kudos [?]: 72876 [3]
Given Kudos: 18863
Location: United States (WA)
GMAT 1: 750 Q49 V42
GPA: 3
Send PM
Re: M01-08 [#permalink]
3
Kudos
Expert Reply
CircleOfLife wrote:
Since this is a PS question. shouldn't the answer choices be either ascending or descending?


You mean to make it easier for you to answer the question?
Good try.... :lol:

P.S. On the GMAT if a question asks for the largest/smallest value, the answer choices do NOT follow a pattern.
Intern
Intern
Joined: 26 Feb 2018
Posts: 41
Own Kudos [?]: 56 [1]
Given Kudos: 43
Location: India
GMAT 1: 640 Q45 V34
GPA: 3.9
WE:Web Development (Computer Software)
Send PM
Re: M01-08 [#permalink]
1
Kudos
I think this is a high-quality question and I agree with explanation. A good question with a little trick to follow.
Math Expert
Joined: 02 Sep 2009
Posts: 92912
Own Kudos [?]: 618902 [0]
Given Kudos: 81595
Send PM
Re: M01-08 [#permalink]
Expert Reply
I have edited the question and the solution by adding more details to enhance its clarity. I hope it is now easier to understand.
Intern
Intern
Joined: 29 Jul 2023
Posts: 30
Own Kudos [?]: 8 [0]
Given Kudos: 8
Location: United States (OH)
GPA: 3.79
Send PM
Re: M01-08 [#permalink]
How I solved it:

\(1^9^9^9\) = 1

When looking at the remaining values, notice that all of the exponents share a GCF: 50. Raise each of these remaining values to the \(\frac{1}{50}\) power

You're now left with \(2^6\), \(3^4\), \(4^2\), and \(16^1\). From here, it's really simple to find each of these values and realize the largest value is \(3^4\) = 81.
Intern
Intern
Joined: 04 Mar 2023
Posts: 1
Own Kudos [?]: 0 [0]
Given Kudos: 4
Send PM
Re M01-08 [#permalink]
I think this is a high-quality question and I agree with explanation.
GMAT Club Bot
Re M01-08 [#permalink]
Moderator:
Math Expert
92912 posts

Powered by phpBB © phpBB Group | Emoji artwork provided by EmojiOne